Archivo de Enunciados • Competencias Internacionales • PAGMO • 2022


Problema 1
Leticia tiene un tablero de $9\times 9$ casillas. Se dice que dos casillas son amigas si comparten un lado o si están en una misma columna, pero en extremos opuestos, o si están en una misma fila, pero en extremos opuestos. De esta forma, cada casilla tiene exactamente $4$ casillas amigas.
Leticia va a pintar cada casilla de uno de tres colores: verde, azul o rojo. Una vez que todas las casillas estén pintadas, en cada casilla va a escribir un número, siguiendo las siguientes reglas:
  • Si la casilla es verde, escribe la cantidad de casillas rojas amigas más dos veces la cantidad de casillas azules amigas.
  • Si la casilla es roja, escribe la cantidad de casillas azules amigas más dos veces la cantidad de casillas verdes amigas.
  • Si la casilla es azul, escribe la cantidad de casillas verdes amigas más dos veces la cantidad de casillas rojas amigas.
Encuentra el máximo valor posible de la suma de los números asignados a las casillas que Leticia pude obtener, sabiendo que ella puede escoger la coloración de las casillas del tablero.

Problema 2
Encontrar todas las tripletas $(p,q,r)$ de enteros positivos que cumplan que $p$ y $q$ son números primos (no necesariamente distintos), que $r$ es par y que$$p^3+q^2=4r^2+45r+103.$$

Problema 3
Sea $ABC$ un triángulo acutángulo con $AB<AC$. Sobre el segmento $BC$ se eligen puntos $P$ y $Q$ tales que $\angle BAP=\angle CAQ<\frac{\angle BAC}{2}$. $B_1$ es un punto en el segmento $AC$. $BB_1$ interseca a $AP$ y $AQ$ en $P_1$ y $Q_1$, respectivamente. Las bisectrices de $\angle BAC$ y $\angle CBB_1$ se cortan en $M$. Si $PQ_1\perp AC$ y $QP_1\perp AB$, demuestra que $AQ_1MPB$ es cíclico.

Problema 4
Sea $ABC$ un triángulo con $AB\neq AC$. Sean $O_1$ y $O_2$ los centros de las circunferencias $\omega _1$ y $\omega _2$ de diámetros $AB$ y $BC$, respectivamente. Sea $P$ un punto en el segmento $BC$ tal que $AP$ interseca a $\omega _1$ en el punto $Q$, con $Q\neq A$. Demostrar que los puntos $O_1$, $O_2$ y $Q$ son colineales si y sólo si $AP$ es la bisectriz del ángulo $\angle BAC$.

Problema 5
Hallar todos los enteros positivos $k$ para los cuales existen enteros positivos $a$, $b$ y $c$ tales que$$|(a-b)^3+(b-c)^3+(c-a)^3|=3\cdot 2^k.$$Nota: Para cualquier número real $x$, el valor absoluto de $x$ se denota por $|x|$ y se define como:$$|x|=\left \{\begin{matrix}x & \text{si} & x\geq 0, \\ -x & \text{si} & x<0.\end{matrix}\right .$$Por ejemplo, $|-3|=3$.

Problema 6
Ana y Bety juegan un juego por turnos de manera alternada. Inicialmente Ana elige un entero positivo impar y compuesto $n$ tal que $2^j<n<2^{j+1}$, con $2<j$. En su primer turno, Bety elige un entero positivo impar y compuesto $n_1$ tal que$$n_1\leq \frac{1^n+2^n+\cdots +(n-1)^n}{2(n-1)^{n-1}}.$$Luego, en su turno, Ana elige un número primo $p_1$ que divida a $n_1$. Si el primo que eligió Ana es $3$, $5$ o $7$, Ana gana, de lo contrario Bety elige un entero positivo impar y compuesto $n_2$ tal que$$n_2\leq \frac{1^{p_1}+2^{p_1}+\cdots +(p_1-1)^{p_1}}{2(p_1-1)^{p_1-1}}.$$Después de eso, en su turno, Ana elige un primo $p_2$ que divida a $n_2$, si $p_2$ es $3$, $5$ o $7$, Ana gana, de lo contrario el proceso se repite. Además, Ana gana en cualquier momento si Bety no puede elegir un entero positivo impar y compuesto en el rango correspondiente. Bety gana si logra jugar al menos $j-1$ turnos. Encontrar cuál de las dos jugadoras tiene estrategia ganadora.